Magnetische Dirac-Monopole und fraktionierte elektrische Ladungsquantisierung durch Quarks

Wenn man die Dirac-Quantisierungsregel für elektrische und magnetische Ladung anwendet, gehe ich davon aus, dass man elektrische Einheitsladungen wie Elektronen betrachtet. Wie gilt die Dirac-Quantisierungsregel für die gebrochenen elektrischen Ladungen von Quarks?

Ich beantworte diese Frage unter physical.stackexchange.com/q/268709 .

Antworten (2)

Die Dirac-Quantisierungsregel ergibt sich aus der Integration des Drehimpulses des überlagerten elektromagnetischen Feldes einer Ladung und eines Monopols. Dieser Drehimpuls erweist sich als endlich und entfernungsunabhängig H zwischen Ladung und Monopol. Das Argument lautet dann, wenn es möglich ist, eine einzelne Grundladung zu isolieren e und einem einzigen fundamentalen Monopol G in einer Region des Weltraums, dann muss der Gesamtdrehimpuls in dieser Region ein Vielfaches von sein . Hier bedeutet "Isolation", dass der Abstand zu jedem anderen Teilchen besteht H .

Beachten Sie jedoch, dass die Isolierung der Partikel kritisch ist. Wenn Sie einen Monopol in die Nähe eines Wasserstoffatoms bringen, verschwindet der Gesamtdrehimpuls des elektromagnetischen Felds, weil die Drehimpulsdichte von der Form ist E × B , die in den Feldern bilinear ist.

Da Quarks beschränkt sind, kann das Argument niemals auf ein Quark angewendet werden.

Das ist ein guter Punkt. Nichtsdestotrotz gilt das Argument für QFTs, bei denen die Anzahl der Geschmacksrichtungen hoch genug ist, dass es nicht zu einer Eindämmung kommt.
Was wäre, wenn Sie die Temperatur über die Hagedorn-Temperatur anheben würden ? Dann würden Quarks dekonfinieren, und Sie könnten ein einzelnes Quark um den Dirac-String transportieren.
Ihr Argument gilt nicht für den Fall, dass Sie ein Quark um einen Monopol in einem Kreis bewegen, der kleiner als der Begrenzungsradius oder oberhalb des Hagedorn-Radius ist. Siehe physical.stackexchange.com/questions/268709/… für die vollständige Antwort.

Die Logik ist dieselbe: Wenn Q ist eine elektrische Ladung und G ist eine magnetische Ladung, die man haben muss G Q 2 π Z (in lazy theoretiker Einheiten). Wenn es also eine größte magnetische Ladung gibt, dann gibt es notwendigerweise eine kleinste elektrische Ladung. Sie müssen nicht davon ausgehen, dass Sie es mit Einheitsgebühren zu tun haben, um dieses Argument vorzubringen.